The Student Room Group

OCR C4 (not mei) 18th June 2013 revision

Scroll to see replies

Original post by 94Singh
its 3/3x .. f'(x)/f(x)


yeah and when you simplify 3/3x it becomes 1/x
Original post by eggfriedrice
Pretty sure it was a maximum. You differentiate it again and find it's negative. (C1)


Yeah, I got maximum, too. d*2y/dx*2 was <0, which made it a maximum. I think. :s-smilie:
Reply 402
that paper was messed up!!!!:mad:
Original post by h2shin
Stationary points for the trig: (pi/2, 1) (pi/6, 1.5) (5pi/6, 1.5)
Any agreements?

Oh and gradient to left was negative and gradient to right was positive so I put minimum but can't be too sure...


how did you get the first one?
Original post by ihatepeople37
yeah i got them apart from the first one.... oh well


I only got two stationary points, also! :eek: Was there another one?
Reply 405
for the stationary point question was t=-1?
Reply 406
ohhh **** just realized they had to be 3 stationary points I put 1 :angry:
Original post by eggfriedrice
Pretty sure it was a maximum. You differentiate it again and find it's negative. (C1)


Just plotted the function. It was a minimum.
Original post by Dizzy in my Head
Yeah, I got maximum, too. d*2y/dx*2 was <0, which made it a maximum. I think. :s-smilie:


but it said by considering the gradient either side
Original post by louise_234
yeah and when you simplify 3/3x it becomes 1/x

but 1/x is integrated it is lnx not ln3x, or am i being really stupid??
So much volume! :frown:
Reply 411
Original post by a10
ohhh **** just realized they had to be 3 stationary points I put 1 :angry:


Thats what I got as well :/ fml
Okay I'm petrified now, you guys all seemed to find it alright! :frown:

I can't be the only one who thought it was horrific? :frown:
Reply 413
Original post by louise_234
yeah and when you simplify 3/3x it becomes 1/x


yeah .. they differentiate to become the same .. try putting them into your calculator if you have one that can differentiate
Reply 414
This is effectively a preliminary markscheme until Mr M posts his full definitive solutions. Some of my answers may be wrong. Spoilered incase you don't want to see them :smile:

*A few additions have been made to the markscheme* - namely the cartesian equation (forgot to put it in)

Spoiler

(edited 10 years ago)
Original post by Maid Marian
Okay I'm petrified now, you guys all seemed to find it alright! :frown:

I can't be the only one who thought it was horrific? :frown:


Dont worry, so did I and most of my year. Just realised marks here and there missed too :frown:
Original post by Maid Marian
Okay I'm petrified now, you guys all seemed to find it alright! :frown:

I can't be the only one who thought it was horrific? :frown:


No. I agree. I thought is was dreadful. :mad: I'm so angry with myself! I can't wait for someone to post an unofficial mark scheme.
I got t=1 and two stationary points ._.
Original post by eggfriedrice
I got t=1 and two stationary points ._.


As did I. All these conflicting answers! :confused:
Original post by Dizzy in my Head
As did I. All these conflicting answers! :confused:


Yeah, I plotted the graph and there are 2 stationary points. But why?

Quick Reply

Latest

Trending

Trending